Q16

 
jacqueline.niba
Thanks Received: 0
Vinny Gambini
Vinny Gambini
 
Posts: 3
Joined: February 07th, 2011
 
 
 

Q16

by jacqueline.niba Wed Sep 14, 2011 11:32 am

For question 16, why couldn't N go in day 2?
 
chike_eze
Thanks Received: 94
Atticus Finch
Atticus Finch
 
Posts: 279
Joined: January 22nd, 2011
 
This post thanked 1 time.
 
trophy
Most Thanked
 

Re: Q15

by chike_eze Sun Oct 16, 2011 10:39 pm

Question is which CANNOT be 2nd. As you've pointed out, Negotiating can go second.

1) G H
2) P N
3) O T

Persuasion cannot go 2nd because it forces Negotiating to day 3, which forces out Telemarketing. i.e., Telemarketing cannot be scheduled on any day if Persuasion is 2nd.

Why? G and O before T, N and T cannot go together, and G and O cannot go together.
 
ferielchebouli
Thanks Received: 0
Forum Guests
 
Posts: 1
Joined: September 29th, 2011
 
 
 

Re: Q16

by ferielchebouli Thu Nov 03, 2011 8:42 pm

Sorry I still dont understand ... I got the same setup

1) GH
2) PN
3) OT

The way I see it P CAN be the second seminar given on the second day...it still allows T to be on the third day..

am I missing something? Can N not be with P? The way I understood it...once P is given then N could be given and isnt that what number 2 points out.
 
eunjung.shin
Thanks Received: 2
Jackie Chiles
Jackie Chiles
 
Posts: 40
Joined: December 08th, 2011
 
 
 

Re: Q16

by eunjung.shin Fri Jun 08, 2012 2:29 am

ferielchebouli Wrote:Sorry I still dont understand ... I got the same setup

1) GH
2) PN
3) OT

The way I see it P CAN be the second seminar given on the second day...it still allows T to be on the third day..

am I missing something? Can N not be with P? The way I understood it...once P is given then N could be given and isnt that what number 2 points out.



Same question.... I got thesame setup that allows P to be on the second day. Can anyone help?
 
timmydoeslsat
Thanks Received: 887
Atticus Finch
Atticus Finch
 
Posts: 1136
Joined: June 20th, 2011
 
 
trophy
Most Thanked
trophy
First Responder
 

Re: Q16

by timmydoeslsat Fri Jun 08, 2012 12:35 pm

P is not before N if it is true that P is the last conference of the second day. If N is also on Day 2, then you are having N come prior to the P, which is a rule violation.
 
eunjung.shin
Thanks Received: 2
Jackie Chiles
Jackie Chiles
 
Posts: 40
Joined: December 08th, 2011
 
 
 

Re: Q16

by eunjung.shin Mon Jun 11, 2012 11:23 pm

timmydoeslsat Wrote:P is not before N if it is true that P is the last conference of the second day. If N is also on Day 2, then you are having N come prior to the P, which is a rule violation.


Thank you! I think I didn't see that the questions say "which one cannot be the SECOND of the second day" ..totally makes sense now.

just one more question.
The rules doesn't say that whether which one of the long or short one goes first on a certain day(it says one short and one long will be given each day), but how do we know we can't have mixed order of long and short for three days?

for example,

1) Long, short
2) short, long
3) long, short

I automatically thought that either all long ones or all short ones go first, but as I review I see the other possibility that I never thought of.

Thanks for your help!
 
ShehryarB30
Thanks Received: 2
Elle Woods
Elle Woods
 
Posts: 100
Joined: July 07th, 2018
 
 
 

Re: Q16

by ShehryarB30 Wed Jan 02, 2019 8:59 pm

Im confused why B is wrong because in slot 1 g/o can go with h, in slot p can go with n and in slot 3 g/o can go with T. So how is it wrong?
User avatar
 
ohthatpatrick
Thanks Received: 3807
Atticus Finch
Atticus Finch
 
Posts: 4661
Joined: April 01st, 2011
 
 
 

Re: Q16

by ohthatpatrick Fri Feb 15, 2019 2:32 am

You wrote:
Im confused why B is wrong because in slot 1 g/o can go with h, in slot p can go with n and in slot 3 g/o can go with T. So how is it wrong?


This scenario is illegal:
g/o, H | p, N | o/g, T

This one would be okay:
g/o, H | N, p | o/g, T

The 3rd rule says that we can't have N until we've already had P.

Since P can be the 2nd spot on the 2nd day, we can get rid of (B).
The question stem wants someone who cannot go 2nd spot on 2nd day.

Other possible scenarios:
p, N | g/o, H | o/g, T
p, N | H, g/o | o/g, T

From those, we can also see that H, G, and O can be in the 2nd spot of the 2nd day, so we can also eliminate (A), (C), and (E).